www.vorhilfe.de
- Förderverein -
Der Förderverein.

Gemeinnütziger Verein zur Finanzierung des Projekts Vorhilfe.de.
Hallo Gast!einloggen | registrieren ]
Startseite · Mitglieder · Impressum
Forenbaum
^ Forenbaum
Status VH e.V.
  Status Vereinsforum

Gezeigt werden alle Foren bis zur Tiefe 2

Navigation
 Startseite...
 Suchen
 Impressum
Das Projekt
Server und Internetanbindung werden durch Spenden finanziert.
Organisiert wird das Projekt von unserem Koordinatorenteam.
Hunderte Mitglieder helfen ehrenamtlich in unseren moderierten Foren.
Anbieter der Seite ist der gemeinnützige Verein "Vorhilfe.de e.V.".
Partnerseiten
Weitere Fächer:

Open Source FunktionenplotterFunkyPlot: Kostenloser und quelloffener Funktionenplotter für Linux und andere Betriebssysteme
Forum "Integration" - noch ein Integral
noch ein Integral < Integration < Funktionen < eindimensional < reell < Analysis < Hochschule < Mathe < Vorhilfe
Ansicht: [ geschachtelt ] | ^ Forum "Integration"  | ^^ Alle Foren  | ^ Forenbaum  | Materialien

noch ein Integral: Frage (beantwortet)
Status: (Frage) beantwortet Status 
Datum: 00:58 So 04.02.2007
Autor: Bastiane

Hallo nochmal! ;-)

Diesmal folgendes Integral:

[mm] \integral_{-\pi}^{\pi}\sin(mx)\sin(nx)\:dx=\pi\delta_{mn} [/mm]

Nach einem Additionstheorem gilt doch:

[mm] \integral_{-\pi}^{\pi}\sin(mx)\sin(nx)\:dx=\br{1}{2}\integral_{-\pi}^{\pi}\cos(x(m-n))\:dx-\br{1}{2}\integral_{-\pi}^{\pi}\cos(x(m+n))\:dx=[\br{\sin(x(m-n))}{m-n}]_0^{\pi}+[\br{\sin(x(m+n))}{m+n}]_0^{\pi} [/mm]

Nun ist doch aber der Sinus von Vielfachen von [mm] \pi [/mm] gleich 0, also stehen da überall nur Nullen und somit ist das ganze Integral =0!?

Wo ist mein Fehler??

Viele Grüße
Bastiane
[cap]



        
Bezug
noch ein Integral: Antwort
Status: (Antwort) fertig Status 
Datum: 01:08 So 04.02.2007
Autor: Marc

Hallo Bastiane,

> Diesmal folgendes Integral:
>  
> [mm]\integral_{-\pi}^{\pi}\sin(mx)\sin(nx)\:dx=\pi\delta_{mn}[/mm]
>  
> Nach einem Additionstheorem gilt doch:
>  
> [mm]\integral_{-\pi}^{\pi}\sin(mx)\sin(nx)\:dx=\br{1}{2}\integral_{-\pi}^{\pi}\cos(x(m-n))\:dx-\br{1}{2}\integral_{-\pi}^{\pi}\cos(x(m+n))\:dx=[\br{\sin(x(m-n))}{m-n}]_0^{\pi}+[\br{\sin(x(m+n))}{m+n}]_0^{\pi}[/mm]
>  
> Nun ist doch aber der Sinus von Vielfachen von [mm]\pi[/mm] gleich
> 0, also stehen da überall nur Nullen und somit ist das
> ganze Integral =0!?
>  
> Wo ist mein Fehler??

Kein Fehler, nur hast Du so nur den Fall [mm] $m\not=n$ [/mm] betrachtet, denn das letzte Gleichheitszeichen gilt natürlich nicht für $m=n$.
Falls $m=n$, ist das Integral wie angegeben [mm] $\pi$ [/mm]

Viele Grüße,
Marc

Bezug
        
Bezug
noch ein Integral: Idee
Status: (Antwort) fertig Status 
Datum: 01:48 So 04.02.2007
Autor: VNV_Tommy

Hallo Bastiane!

> Hallo nochmal! ;-)
>  
> Diesmal folgendes Integral:
>  
> [mm]\integral_{-\pi}^{\pi}\sin(mx)\sin(nx)\:dx=\pi\delta_{mn}[/mm]
>  
> Nach einem Additionstheorem gilt doch:
>  
> [mm]\integral_{-\pi}^{\pi}\sin(mx)\sin(nx)\:dx=\br{1}{2}\integral_{-\pi}^{\pi}\cos(x(m-n))\:dx-\br{1}{2}\integral_{-\pi}^{\pi}\cos(x(m+n))\:dx=[\br{\sin(x(m-n))}{m-n}]_0^{\pi}+[\br{\sin(x(m+n))}{m+n}]_0^{\pi}[/mm]
>  

Stammfunktion stimmt soweit ich das überblicke.

> Nun ist doch aber der Sinus von Vielfachen von [mm]\pi[/mm] gleich
> 0, also stehen da überall nur Nullen und somit ist das
> ganze Integral =0!?

Der Sinus von ganzzahligen [mm] \pi [/mm] ist mit Sicherheit gleich Null. Ich denke es kommt nun entscheidend auf den Definitionsbereich von m und n um weiter zu argumentieren. Sofern mit m-n und m+n auch nicht ganzzahlige Ergebnisse gebildet werden können, sollte auch das (m-n)-fache bzw. (m+n)-fache von [mm] \pi [/mm] ungleich Null sein. Wenn z.B. m-n=0,5 ist, dann wäre der [mm] sin(0,5\pi) [/mm] gesucht, der sich ja bekanntlich zu 1 ergibt.

Soweit meine Idee dazu. Vielleicht kannst du ja was damit anfangen.

Gruß,
Tommy

Bezug
                
Bezug
noch ein Integral: Danke.
Status: (Mitteilung) Reaktion unnötig Status 
Datum: 16:43 So 04.02.2007
Autor: Bastiane

Hallo ihr beiden!

Vielen Dank, dass ihr euch meine eigentlich viel zu simple Frage durchgelesen und darauf geantwortet habt. :-) Ich hatte tatsächlich den Fall $m=n$ total vergessen. Allerdings habe ich jetzt dafür direkt am Anfang ein anderes Additionstheorem angewendet - gestern abend dachte ich, man könnte es auch direkt sehen oder so?

Und m und n sind wohl schon natürliche Zahlen, deswegen brauche ich keine weitere "Fallunterscheidung" zu machen. :-)

Viele Grüße
Bastiane
[cap]


Bezug
Ansicht: [ geschachtelt ] | ^ Forum "Integration"  | ^^ Alle Foren  | ^ Forenbaum  | Materialien


^ Seitenanfang ^
ev.vorhilfe.de
[ Startseite | Mitglieder | Impressum ]